aileenann
Thanks Received: 227
Atticus Finch
Atticus Finch
 
Posts: 300
Joined: March 10th, 2009
 
This post thanked 1 time.
 
 

Q14 - It has been suggested that

by aileenann Fri Dec 31, 1999 8:00 pm

This is the fun but unusual question type where the LSAT asks us to be a bit formal in our logical analysis and actually say what role a particular portion of the text plays in the overall argument. As a general rule of thumb, I am wary of answers that are overly complicated or hard to understand - I only pick these if all of the other answers are definitely wrong.

Also, I find it helpful to find the conclusion and then relate the statement they are asking about to the conclusion - specifically does it help or hurt the conclusion, or is it the conclusion. Let's try to do this here.

Conclusion: It would be too simple to let market forces determine what shows on television.

The claim that tv is important politically and culturally tends to support the author's conclusion. With that in mind, let's check out our options.

(A) doesn't work - this wouldn't describe something that is in sync with the conclusion.

(B) is the opposite of what we want - it's offered against the market forces idea, not for it.

(C) is the opposite of what we want - it's offered in opposition to the idea that market forces alone should determine the content on television.

(D) is the answer. We know already that this statement supports the ultimate conclusion, which this answer choice affirms. There is a lot more going on, but in a fast sweep let's just keep it for now and look at (E).

(E) seems backwards. The idea that something is important does not support the idea that it is used often. Rather it's the opposite - when something is used often it is important. So this is probably not the answer, but it should give you some insight into why (D) is the answer.

If this were the real LSAT, I'd probably cross out (E) and pick (D) without thinking about it anymore. After all, this is a very time pressured test. But let's take a second to go back and look at the rest of the jumble of answer (D).

Let's think about why this is an intermediate conclusion. It's because something being important is not clearly a premise - it would seem to require additional proof to show that something is important. That is precisely the role the other facts in the second part of the argument play overall - they make this intermediate conclusion feasible, which in turn holds up the author's ultimate conclusion.

This is a tough problem and worth thinking about a little extra. Does anyone have any questions or other comments to make about the problem? I'd love to hear what you all are thinking!


#officialexplanation
 
bradleygirard
Thanks Received: 17
Forum Guests
 
Posts: 27
Joined: May 12th, 2010
 
 
trophy
First Responder
 

Re: Q14 - It has been suggested that

by bradleygirard Thu May 20, 2010 11:23 pm

Indeed quite the jumble, and in a timed test, I eliminated all of the other answers, took another quick glance over (d) and with the tiniest bit of skepticism, picked it. The statement that television is important politically and culturally simply seems like a premise upon first read. Now, if we wanted to pick apart an argument of someone who stated that as a simple premise, it wouldn't be too tough, but for the sake of the test, I was searching for an answer that pointed to it being a premise. Both (c) and (e) try to attract us in use of the word premise, but they don't show what it is actually in support of. The next best option is a sub-conclusion or intermediate conclusion, because hey, those are premises too. In the following lines, the author does indeed make an attempt to validate both political and cultural influence, thus leading to this intermediate conclusion.
In a logical sense, this structure seems odd, and can be indeed very confusing, but this we hear arguments like this all the time. I guess that just adds to the confusion of the question.
User avatar
 
ManhattanPrepLSAT1
Thanks Received: 1909
Atticus Finch
Atticus Finch
 
Posts: 2851
Joined: October 07th, 2009
 
This post thanked 4 times.
 
 

Re: PT 53 S 1 Q 14 It has been suggested that a television set

by ManhattanPrepLSAT1 Fri May 21, 2010 3:21 pm

I like the way you broke that down and caught the fact the the claim only "seems" like a premise, rather than "is" a premise.

Let me walk you through my thought process.

The first sentence is an opposing point.
The second sentence denies the opposing point.
The third sentence contains two parts _______(1) since ________(2).
Part 1 is the conclusion and part 2 is the evidence.
The fourth sentence is evidence for why part 2 is true.
The fifth sentence is evidence for why part 2 is true.

Conclusion = main point
Subsidiary Conclusion = supported evidence
Premise = unsupported evidence.

So, the last two statements are evidence. The conclusion is that "some governmental control is needed." The subsidiary conclusion is that "television is so important politically and culturally." The two statements after that claim are premises.

The question asks to find the role of the claim that "television is so important politically and culturally."

So, we're looking for "it's a subsidiary conclusion." Answer choices (B) and (D) should be on the radar.

(B) claims that the conclusion is within the opposing point. So is not correct.
(D) has all the parts lined up correctly.
 
cyruswhittaker
Thanks Received: 107
Forum Guests
 
Posts: 246
Joined: August 11th, 2010
 
 
trophy
Most Thanked
trophy
First Responder
 

Re: PT 53 S 1 Q 14 It has been suggested that a television set

by cyruswhittaker Mon Sep 13, 2010 3:02 am

For this question, I have found that sometimes keeping it simple can really help. I think of an intermediate conclusion as being both supported by something else and also supporting something itself, which is exactly what the phrase does.
 
jamiejames
Thanks Received: 3
Atticus Finch
Atticus Finch
 
Posts: 116
Joined: September 17th, 2011
 
 
 

Re: Q14 - It has been suggested that

by jamiejames Mon May 07, 2012 5:43 pm

I that the conclusion was "some governmental control is needed." I was still able to get the correct answer, but could someone explain why "but this approach is too simple" is the main conclusion?
 
timmydoeslsat
Thanks Received: 887
Atticus Finch
Atticus Finch
 
Posts: 1136
Joined: June 20th, 2011
 
 
trophy
Most Thanked
trophy
First Responder
 

Re: Q14 - It has been suggested that

by timmydoeslsat Tue May 08, 2012 3:57 pm

I see this one as:

(1) Main conclusion: The approach of market forces determining what is seen on TV is too simple.

(2) Intermediate conclusion: Some governmental control is needed.

(3) Intermediate conclusion: Television is so important politically and culturally.

It looks like this visually:

Premise support at the bottom of the stimulus ---> IC (3) ---> IC (2) ---> Main Conclusion
 
nthakka
Thanks Received: 6
Vinny Gambini
Vinny Gambini
 
Posts: 25
Joined: March 13th, 2013
 
This post thanked 1 time.
 
 

Re: Q14 - It has been suggested that

by nthakka Sat Aug 17, 2013 4:35 pm

Keeping it simple on these questions is the best approach.

(A): Clearly, this isn't something the argument is trying to discredit. Eliminate.

(B): It is an intermediate conclusion, but it isn't supporting the claim of "TV should be thought of as nothing more than a 'toaster with pictures'" how is this a claim supported in the stimulus? Eliminate. If anything, the argument is trying to go against this opposite claim brought about in the 1st sentence.

(C): Author isn't supporting this claim, but trying to discredit it.

(D): Bingo. The major tip off for this AC is if you can realize that "some government control of TV" is a claim. All the other ACs that have "claims" in it, aren't actually claims in the stimulus. Rather they are opposite points, premises, etc. Read carefully.

(E): TV as the primary medium isn't the claim made in the stimulus. That sentence is a premise offered in support of the intermediate conclusion of why TV is so important culturally/politically. So this AC has it backwards.
 
cstars808
Thanks Received: 0
Vinny Gambini
Vinny Gambini
 
Posts: 4
Joined: August 07th, 2015
 
 
 

Re: Q14 - It has been suggested that

by cstars808 Wed Mar 15, 2017 3:51 pm

under timed conditions, i picked e because when I was looking at the sentence it started with " since" as since is a premise indicator. how can we determine that the sentence in question is the intermediate conclusion without diagramming? because it wasn't until i diagrammed it out that i realized it was the intermediate conclusion.
User avatar
 
ohthatpatrick
Thanks Received: 3808
Atticus Finch
Atticus Finch
 
Posts: 4661
Joined: April 01st, 2011
 
This post thanked 2 times.
 
 

Re: Q14 - It has been suggested that

by ohthatpatrick Thu Mar 16, 2017 2:21 pm

Determine the Function questions are famous for testing us on Intermediate Conclusions, so if you don't already think of them with that distinction, that will help.

It's such a common correct answer, that any time you're looking at a supporting idea, you might ask yourself, "Is this merely a premise, or is this technically an intermediate conclusion?"

But mostly I would just go to the answer choices thinking, "it's a supporting idea ... thus I need to check out any answers that say
- premise
- intermediate conclusion
- an illustration of
- an example of
- support for
- evidence for

To judge whether something is an intermediate conclusion, I just use The Why Test.

I say the claim, ask "Why?", and see if I can point to any OTHER claims in the argument that sound like supporting reasons.

If I asked myself, "WHY is TV so important politically and culturally?",
I would then see that the last two sentences unpack this claim and provide supporting ideas.

In terms of picking (E) under timed conditions, you know you shouldn't JUST pick an answer because it says "premise", right?

These answers are almost always structured to say
"It is a premise that is offered in support of the claim that [supposedly the conclusion]"

So we can't pick that answer until we've verified that it correctly identified the conclusion being supported.

Any time half of a sentence is prefaced with "because/since", the OTHER half is a conclusion being supported by the because/since claim.

So we know that our claim is offered in support of the idea that "Some governmental control is needed".
 
cstars808
Thanks Received: 0
Vinny Gambini
Vinny Gambini
 
Posts: 4
Joined: August 07th, 2015
 
 
 

Re: Q14 - It has been suggested that

by cstars808 Thu Mar 16, 2017 4:23 pm

yes I understand thank you for clarifying
 
AbrahamS97
Thanks Received: 0
Vinny Gambini
Vinny Gambini
 
Posts: 4
Joined: August 14th, 2017
 
 
 

Re: Q14 - It has been suggested that

by AbrahamS97 Sun Sep 03, 2017 5:41 pm

Under timed conditions, it seems understanding the distinction between a premise and an intermediate conclusion (and being able to identify them quickly) is the best way to go on this. It's not the kind of question you want to spend more than a minute on.
 
at9037
Thanks Received: 0
Vinny Gambini
Vinny Gambini
 
Posts: 14
Joined: September 08th, 2015
 
 
 

Re: Q14 - It has been suggested that

by at9037 Sat Dec 23, 2017 8:02 pm

If there was a question that asked “What role is played by the statement some governmental control is needed?”, and there was an answer choice that said : “It is an intermediate conclusion offered in support of the main conclusion and supported by the claim that television is important polictically and culturally”, would this be correct?
User avatar
 
ohthatpatrick
Thanks Received: 3808
Atticus Finch
Atticus Finch
 
Posts: 4661
Joined: April 01st, 2011
 
 
 

Re: Q14 - It has been suggested that

by ohthatpatrick Tue Jan 02, 2018 3:35 pm

(sorry for the holiday-delayed response)

Yeah, I think that would be accurate, although you're saying that "some govt control is needed" is intermediate, and the main conclusion is "that approach is too simple"?

I think LSAT would have a hard time granting one of those as the correct interpretation and the other wrong.

Certainly, the author's main purpose seems to be directed at "some govt control is needed", so I'd be more tempted to call that the main point.

But you could make a case for either one of these:
MAIN CONC - Some govt control is needed
why?
INTERMEDIATE - because the alternative approach is too simple

or
MAIN CONC - the alternative approach is too simple
why?
INTERMEDIATE - because some govt control is needed
 
MingL143
Thanks Received: 0
Vinny Gambini
Vinny Gambini
 
Posts: 23
Joined: September 15th, 2018
 
 
 

Re: Q14 - It has been suggested that

by MingL143 Tue Sep 18, 2018 6:33 pm

ohthatpatrick Wrote:Determine the Function questions are famous for testing us on Intermediate Conclusions, so if you don't already think of them with that distinction, that will help.

It's such a common correct answer, that any time you're looking at a supporting idea, you might ask yourself, "Is this merely a premise, or is this technically an intermediate conclusion?"

But mostly I would just go to the answer choices thinking, "it's a supporting idea ... thus I need to check out any answers that say
- premise
- intermediate conclusion
- an illustration of
- an example of
- support for
- evidence for

To judge whether something is an intermediate conclusion, I just use The Why Test.

I say the claim, ask "Why?", and see if I can point to any OTHER claims in the argument that sound like supporting reasons.

If I asked myself, "WHY is TV so important politically and culturally?",
I would then see that the last two sentences unpack this claim and provide supporting ideas.

In terms of picking (E) under timed conditions, you know you shouldn't JUST pick an answer because it says "premise", right?

These answers are almost always structured to say
"It is a premise that is offered in support of the claim that [supposedly the conclusion]"

So we can't pick that answer until we've verified that it correctly identified the conclusion being supported.

Any time half of a sentence is prefaced with "because/since", the OTHER half is a conclusion being supported by the because/since claim.

So we know that our claim is offered in support of the idea that "Some governmental control is needed".


Could you please explain the answer choice "A"? I got this question correct but I couldn't give the exact explanation why A is wrong.( This might be because of my language capability-I am not a native-English speaker. I eliminated "A" because of the guts feeling and because the choice D is perfect to describe the function of the sentence ).
Is A wrong because of "view"? but isn't the intermedia conclusion can be interpreted as the author's view, the view to oppose the first opinion ("let the television freely be")?
Or is A wrong because of "discredit"? If I rewrite the answer A to "It states a view that the argument as a whole is designed to disagree with the first opinion", would "A" be correct?
If you don't agree, could you please give a classic argument example to illustrate choice "A"?
User avatar
 
ohthatpatrick
Thanks Received: 3808
Atticus Finch
Atticus Finch
 
Posts: 4661
Joined: April 01st, 2011
 
 
 

Re: Q14 - It has been suggested that

by ohthatpatrick Thu Oct 04, 2018 1:42 pm

We knew that the claim they're asking about here was one of our author's supporting ideas.

Without even reading the substance of the claims, you can ascertain that merely from the structure.

It has been suggested that X
(so, X is someone else's view)

.... and that since Y is true, we get to Z
(someone else is combining X and Y to get to conclusion Z)

But that approach is too simple
(this is our author's opinion, so we know she disagrees with the aforementioned view)

G is needed, since T.
(G is a conclusion our author has drawn, and T is a supporting reason for our author's conclusion)

The question stem asks us about claim T, which we just said was a claim the author used to support a conclusion she made.

Hence, we'd get rid of (A) the second we see it say "the [author's] argument as a whole is designed to discredit claim T."

Huh? Why would an author be trying to discredit her own premise?

If you're a lawyer, you try to discredit your opponent's witnesses / your opponent's evidence. You don't try to discredit your own evidence (you try to bolster it and corroborate it).

The argument as a whole IS trying to discredit a certain view:
"We can let market forces determine what is seen on television"

An answer like (A) would point to something that came BEFORE a "but/yet/however I disagree" pivot point.

Had the question stem asked about the role played by the claim that "we can let [market forces] determine what is seen on television", then (A) would be the correct answer.

Hope this helps.
 
VendelaG465
Thanks Received: 0
Elle Woods
Elle Woods
 
Posts: 66
Joined: August 22nd, 2017
 
 
 

Re: Q14 - It has been suggested that

by VendelaG465 Tue Nov 13, 2018 7:06 pm

So is it safe to say that intermediate conclusions and premises are the also the same? they seem interchangeable in the answer choices
User avatar
 
ohthatpatrick
Thanks Received: 3808
Atticus Finch
Atticus Finch
 
Posts: 4661
Joined: April 01st, 2011
 
 
 

Re: Q14 - It has been suggested that

by ohthatpatrick Sat Nov 17, 2018 12:44 am

They're usually not interchangeable in terms of how LSAT uses them.

Usually,
PREMISE = unsupported premise
SUBSIDIARY / INTERMEDIATE CONCLUSION = supported premise

But I have seen at least one example of a correct answer on a Role/Function question that called a claim a Premise, rather than an Intermediate Conclusion, even though that Premise definitely had some support.

Technically, a claim that could be called an Intermediate Conclusion is a PREMISE in relation to the MAIN CONCLUSION, but it's also a CONCLUSION in relation to the evidence offered on its behalf.

The moral to the story is this:

if it's being called PREMISE
just verify that it PROVIDES support to some conclusion

if it's being called an INTERMEDIATE CONC / SUBSIDIARY CONC / or "a conclusion, but not the main conclusion"
not only verify that it PROVIDES support to some conclusion
but also verify that it RECEIVES support from at least one explicit idea
 
GabeR312
Thanks Received: 1
Vinny Gambini
Vinny Gambini
 
Posts: 4
Joined: June 04th, 2019
 
 
 

Re: Q14 - It has been suggested that

by GabeR312 Sat Jun 29, 2019 4:10 pm




I am confused here. The sentence "Some government control is needed, since television is so important politically and culturally" is an intermediate conclusion. It supports the MC "But that approach is too simple".

When the Stimulus asks what most accurately expresses the role of "TV is so important politically and culturally", wouldn't the correct answer be that it is the premise of an intermediate conclusion due to the indicator word "since"? I thought that the IC itself contains its own CONCLUSION and a PREMISE. (The Premise in the IC is "Since tv is so important politically and culturally" while the conclusion is "gov government control is needed".)

OR does an IC not have a premise + conclusion?
User avatar
 
ohthatpatrick
Thanks Received: 3808
Atticus Finch
Atticus Finch
 
Posts: 4661
Joined: April 01st, 2011
 
 
 

Re: Q14 - It has been suggested that

by ohthatpatrick Tue Jul 02, 2019 1:06 pm

The sentence "Some government control is needed, since television is so important politically and culturally" is an intermediate conclusion. It supports the MC "But that approach is too simple".

A couple issues with this.

1. "Some government control is needed, since TV is so important politically and culturally" is a conclusion and a premise. You're acting like it's just one claim, one ingredient.

2. We don't necessarily agree that the MC is "But that approach is too simple".
It works logically both ways, whether you say
"Letting market forces determine is too simple, since some govt control is needed"
or
"Some govt control is needed, since letting market forces determine is too simple"

The answer choices stay agnostic in regards to what claim is the MC.


When the Stimulus asks what most accurately expresses the role of "TV is so important politically and culturally", wouldn't the correct answer be that it is the premise of an intermediate conclusion due to the indicator word "since"?


Because of the indicator word "since", we know for sure that this claim supports the idea that "some govt control is needed".

So you'd be looking for PREMISE or INTERMEDIATE CONC, depending on whether this 'since' claim is unsupported or supported, respectively.


I thought that the IC itself contains its own CONCLUSION and a PREMISE. (The Premise in the IC is "Since tv is so important politically and culturally" while the conclusion is "gov government control is needed".)

OR does an IC not have a premise + conclusion?


This seems like terminological confusion. A conclusion can't contain a conclusion and a premise. I think you're confusing conclusion with the word 'argument', which contains a minimum of two claims: a conclusion and a premise.

A conclusion is always just one claim.

When you call a claim the Main Conclusion, you imply that other supporting claims were all ultimately aimed at supporting this MC claim.

When you call a claim an IC, you are saying that this claim was supported by at least one idea (which is what makes it a conclusion) and that this claim helps to support some other claim (which is what makes it intermediate).

Hope this helps.